Switch to: References

Add citations

You must login to add citations.
  1. A model of $$\mathsf {ZFA}+ \mathsf {PAC}$$ ZFA + PAC with no outer model of $$\mathsf {ZFAC}$$ ZFAC with the same pure part.Paul Larson & Saharon Shelah - 2018 - Archive for Mathematical Logic 57 (7-8):853-859.
    We produce a model of \ such that no outer model of \ has the same pure sets, answering a question asked privately by Eric Hall.
    No categories
    Direct download (2 more)  
     
    Export citation  
     
    Bookmark  
  • The Vaught Conjecture: Do Uncountable Models Count?John T. Baldwin - 2007 - Notre Dame Journal of Formal Logic 48 (1):79-92.
    We give a model theoretic proof, replacing admissible set theory by the Lopez-Escobar theorem, of Makkai's theorem: Every counterexample to Vaught's Conjecture has an uncountable model which realizes only countably many ℒ$_{ω₁,ω}$-types. The following result is new. Theorem: If a first-order theory is a counterexample to the Vaught Conjecture then it has 2\sp ℵ₁ models of cardinality ℵ₁.
    Direct download (5 more)  
     
    Export citation  
     
    Bookmark   1 citation  
  • Knight's model, its automorphism group, and characterizing the uncountable cardinals.Greg Hjorth - 2002 - Journal of Mathematical Logic 2 (01):113-144.
    We show that every ℵα can be characterized by the Scott sentence of some countable model; moreover there is a countable structure whose Scott sentence characterizes ℵ1 but whose automorphism group fails the topological Vaught conjecture on analytic sets. We obtain some partial information on Ulm type dichotomy theorems for the automorphism group of Knight's model.
    Direct download (4 more)  
     
    Export citation  
     
    Bookmark   14 citations  
  • The Nonabsoluteness of Model Existence in Uncountable Cardinals for $L{omega{1},omega}$.Sy-David Friedman, Tapani Hyttinen & Martin Koerwien - 2013 - Notre Dame Journal of Formal Logic 54 (2):137-151.
    For sentences $\phi$ of $L_{\omega_{1},\omega}$, we investigate the question of absoluteness of $\phi$ having models in uncountable cardinalities. We first observe that having a model in $\aleph_{1}$ is an absolute property, but having a model in $\aleph_{2}$ is not as it may depend on the validity of the continuum hypothesis. We then consider the generalized continuum hypothesis context and provide sentences for any $\alpha\in\omega_{1}\setminus\{0,1,\omega\}$ for which the existence of a model in $\aleph_{\alpha}$ is nonabsolute . Finally, we present a complete (...)
    Direct download (4 more)  
     
    Export citation  
     
    Bookmark   1 citation